LSAT and Law School Admissions Forum

Get expert LSAT preparation and law school admissions advice from PowerScore Test Preparation.

User avatar
 Dave Killoran
PowerScore Staff
  • PowerScore Staff
  • Posts: 5852
  • Joined: Mar 25, 2011
|
#43587
Complete Question Explanation
(The complete setup for this game can be found here: lsat/viewtopic.php?t=8007)

The correct answer choice is (C)

If X and Z are selected as the two plants, it follows that W and Y are not selected (because only two of the four plants are selected). If W is not selected, then J cannot be selected. Thus, any answer choice which contains J must be incorrect. Accordingly, answer choices (B), (D), and (E) can be eliminated. Answer choice (A) can be eliminated since it contains both G and H, a violation of the first rule. Thus, answer choice (C) is correct.

Get the most out of your LSAT Prep Plus subscription.

Analyze and track your performance with our Testing and Analytics Package.